[Date Prev][Date Next][Thread Prev][Thread Next][Date Index][Thread Index]

Re: [obm-l] integral



Aqui vai uma outra solução bem interessante para a integral I = int(0-->+00) (arctan(pi.x) - arctan(x))/x dx.

Ela se baseia na observação de que arctan(pi.x) - arctan(x) eh a integral de 1/(t^2+1) de x até pi.x (*).

Logo, a integral pedida pode ser calculada como um integral dupla:

I = Integral Dupla_x=0..oo_t=x..pix_(dtdx/(t^2+1)/x)

Trocando a ordem de integracao,

I = Integral Dupla_t=0..oo_x=t/pi..t_(dxdt/(t^2+1)/x)

E agora é fácil, pois Integral_x=t/pi..t_(dx/x) = lnt-ln(t/pi) = ln(pi) eh constante, implicando

I = ln(pi)*Integral_t=0..oo_(dt/(t^2+1)) = ln(pi)*pi/2 pela observacao *.

Abracos,
Marcio Cohen

On 4/5/07, Demetrio Freitas <demetrio_freitas_2002_10@yahoo.com.br> wrote:

Buenas,

Vamos começar pela fórmula da integral por partes:

int(a..b)(u dv) = uv(b)-uv(a) -int(a..b)(v du)

No caso, temos:
u = arctan(pi.x) - arctan(x)
v = ln(x)

int(0..+oo)( (arctan(pi.x ) - arctan(x) )/x dx =

lim(x->oo)( (arctan(pi.x) - arctan(x) )*ln(x) -

lim(x->0)( (arctan(pi.x) - arctan(x)  )*ln(x) -

int(0..oo)( (Pi/(1+Pi^2*x^2)-1/(1+x^2) )*ln(x)) dx

O segundo limite é zero (basta olhar a expansão de
taylor para arctan(x)).

O primeiro limite também é zero. Uma forma de ver pode
ser:

(arctan(pi.x) - arctan(x))*ln(x) =  (arctan(pi.x) -
arctan(x))  /  (1/ln(x))

lim(x->oo) ((arctan(pi.x) - arctan(x)) / (1/ln(x)) =
LHospital =>
lim(x->oo) (( Pi/(1+Pi^2*x^2) - 1/(1+x^2) ) *
ln(x)^2*x )  = 0

Então ficamos com:

int(0..+oo)( ( arctan(pi.x) - arctan(x) )/x dx =
int(0..oo)( -f(x) ) dx, Onde:
f(x) = ( Pi/(1+Pi^2*x^2) - 1/(1+x^2) ) * ln(x)

Agora vamos considerar a integral tomada entre -oo e
+oo, e lembrar que, para x E R>0, ln(-x)=ln(x)+i*Pi.

Assim:
int(-oo..+oo)(f(x)) dx = 2*int(0..oo) f(x)  dx +
i*Pi*int(0..oo) Pi/(1+Pi^2*x^2) -1/(1+x^2)) dx

Bem,  int(-oo..+oo) f(x)  dx pode ser calculada por
resíduos. Depois, vc toma a parte real para calcular
int(0..oo) f(x)  dx =>

f(x) tem dois pólos no semiplano complexo z=x+i*y com
y>0, que são: z=i e z=i/Pi.

Res(z=i) = lim( x->i ) ( f(x)*(x-i) ) = -Pi/4
Res(z=i/Pi) = lim( x->i/Pi )( f(x)*(x-i/Pi) ) = Pi/4
+i/2*ln(Pi)

int(-oo..+oo) f(x) dx =2*Pi*i*( -Pi/4+Pi/4+i/2*
ln(Pi))

int(-oo..+oo) f(x) dx = -Pi*ln(Pi)

A integral pedida é então:

int(0..+oo) (arctan(pi.x) - arctan(x))/x dx =

int(0..oo) -f(x) dx = 1/2 * Pi * ln(Pi)

[]´s Demetrio



--- BRENER < CARLOSBRENER@terra.com.br> escreveu:

> Ola, gostaria de uma ajudinha na integral
>
> int(0-->+00) (arctan(pi.x) - arctan(x))/x dx